Đến nội dung

Hình ảnh

Marathon số học Olympic

- - - - -

  • Please log in to reply
Chủ đề này có 191 trả lời

#141
H S G S

H S G S

    Lính mới

  • Thành viên mới
  • 5 Bài viết

Bạn HSGS trình bày lại được không , mình không hiểu bạn định làm gì viết cũng khó hiểu nữa . Nếu chơi " đao to mổ gà " thì dùng định lý mahailescu =)) còn không thì mình nhớ anh IMOer đã giải với $p=5$ và chắc là tương tự thôi .

Chắc ý bạn là mihailescu :v nhưng ko có chứng minh sơ cấp, với cả 2 bài này ý tưởng khác nhau :v Đừng nhầm lẫn là tương tự nha

P/s: Anh IMOer ơi lời giải bài 49 ...


Bài viết đã được chỉnh sửa nội dung bởi H S G S: 11-06-2016 - 06:17


#142
IMOer

IMOer

    Hạ sĩ

  • Thành viên
  • 50 Bài viết

Em nghĩ dãy $k_{n}$ phải nhỏ hơn $10$ và cách chúng ta thêm vào phải lặp lại em không chắc về ý kiến của mình . Mong anh có ý kiến thêm .

Lời giải bài này yêu cầu $k_n$ phải khác 9 (Cũng có thể nếu $k_n$ có thể bằng 9 thì kết luận trên vẫn đúng).

Còn $k_n$ có thể tuỳ ý, không cần lặp lại, ví dụ: $a_1=2016; a_2=20162; a_3=201621; a_4=2016201; a_5=20162010;...$ là một dãy hợp lệ.



#143
IMOer

IMOer

    Hạ sĩ

  • Thành viên
  • 50 Bài viết

Chắc ý bạn là mihailescu :v nhưng ko có chứng minh sơ cấp, với cả 2 bài này ý tưởng khác nhau :v Đừng nhầm lẫn là tương tự nha

P/s: Anh IMOer ơi lời giải bài 49 ...

Tặng các bạn lời giải bài 49:

Xét số nguyên tố $r$ bất kỳ.

Ta gọi $p\in S$ là “tốt” nếu tồn tại vô số phần tử thuộc $S$ đồng dư $p$ mod $r$.

Từ đó dẫn tới có hữu hạn số nguyên tố $p\in S$ là “không tốt”. Gọi $a$ là tích tất cả các số “không tốt” trong $S$ và $b$ là tích của một số số “tốt” bất kỳ trong $S$, khi đó mọi ước nguyên tố của $ab+1$ đều là “tốt”.

Ta xây dựng dãy số $\left( {{x}_{n}} \right)$ như sau: ${{x}_{0}}=1$. Giả sử $x{{y}_{n-1}}+1={{p}_{1}}{{p}_{2}}...{{p}_{m}}$ thì ${{p}_{i}}$ đều là các số “tốt”, nên ta có thể chọn ${{q}_{i}}$ đôi một phân biệt sao cho ${{q}_{i}}\equiv {{p}_{i}}\ \left( \bmod \ r \right)$, khi đó ta lấy: ${{x}_{n}}={{q}_{1}}{{q}_{2}}...{{q}_{m}}$.

Chứng minh bằng quy nạp ta được: ${{x}_{n}}\equiv {{a}^{n}}+...+a+1\ \left( \bmod \ r \right)$.

Dựa trên cách xây dựng dãy $\left( {{x}_{n}} \right)$ ta sẽ chứng minh $r\in S$.

Nếu $a\equiv 0\ \left( \bmod \ r \right)$ thì hiển nhiên $r\in S$ theo cách xây dựng $a$.

Nếu $a\equiv 1\ \left( \bmod \ r \right)$ thì ta có: $r|{{x}_{r-1}}$ nên $r\in S$ theo cách xây dựng ${{x}_{r-1}}$.

Nếu $a\not{\equiv }0\ \left( \bmod \ r \right)$ và $a\not{\equiv }1\ \left( \bmod \ r \right)$ thì ta có: $\left( a-1 \right){{x}_{r-2}}\equiv {{x}^{r-1}}-1\equiv 0\ \left( \bmod \ r \right)$, nên $r\in S$ theo cách xây dựng ${{x}_{r-2}}$.

Vậy $r\in S$ với $r$ là số nguyên tố bất kỳ.



#144
bangbang1412

bangbang1412

    Độc cô cầu bại

  • Phó Quản lý Toán Cao cấp
  • 1667 Bài viết
Lời giải bài 54.
Ta giả sử phản chứng, tức là tồn tại một dãy như đề bài sao cho chỉ có vô hạn số nguyên tố. Tức là từ $N_{0}$ nào đó trở đi thì $a_{n}$ là số nguyên tố. Ta đặt $b_{1} = a_{N_{0}}, b_{n + 1} = b_{n}\times 10 + u_{n}$ với $0 < u_{n} < 9$, dãy này chỉ gồm toàn số nguyên tố.
Bây giờ để ý nếu có $u_{n}$ nào đó thỏa $u_{n} \in \{0, 2, 4, 5, 6, 8\}$ sẽ mâu thuẫn với giả thiết của ta. Vì vậy nên $u_{n} \in \{1, 3, 7\}$.
i) Nếu $1$ hoặc $7$ được sử dụng vô hạn lần. Để ý là $1 \equiv 7\pmod{3}$ nên ta xem chúng là như nhau, thì sau ba lần liên tiếp sử dụng ta thu được ba số 'liên tiếp modulo 3' của dãy. Tức là sẽ có một số chia hết cho $3$. Vô lý.
ii) Vì vậy sẽ chỉ có $3$ được sử dụng vô hạn lần. Tức là tồn tại $A_{0}$ sao cho với mọi $n > A_{0}$, $u_{n} = 3$. Nếu $b_{A_{0}}$ chia hết cho $3$ tức là $b_{A_{0}} = 3$ do $b_{n}$ là dãy nguyên tố. Suy ra $b_{A_{0} + 1} = 33$ là hợp số, vô lý.
Vì vậy nên $b_{A_{0}}$ không là bội của $3$. Gọi $p$ là ước nguyên tố của $b_{A_{0}}$ với $p \neq 2; 3; 5$:
Lúc này ta thu được tồn tại phần tử của dãy $b_{n}$ có dạng $b_{A_{0}}\times 10^{k(p - 1)} + \frac{10^{k(p - 1)} - 1}{3} \vdots p$. Vô lý.
=)) Anh IMOer thấy cái này quen không anh ^_^ - Bằng .
Bài 55 : Với $n = \prod_{i=1}^{k}p_{i}^{\alpha_{i}}$ trong đó $p_{i} \in P$ và khác nhau . Đặt $f(n)=\prod_{i=1}^{k}\alpha_{i}p_{i}^{\alpha_{i}-1}$ . Chứng minh tồn tại vô hạn $n$ mà $f(n)=f(n-1)+1$ .
Dựa trên ý tưởng của Bằng, mình xin phép được sửa đổi tí cho dễ hiểu - Ego.

Bài viết đã được chỉnh sửa nội dung bởi Ego: 12-06-2016 - 19:58

$$[\Psi_f(\mathbb{1}_{X_{\eta}}) ] = \sum_{\varnothing \neq J} (-1)^{\left|J \right|-1} [\mathrm{M}_{X_{\sigma},c}^{\vee}(\widetilde{D}_J^{\circ} \times_k \mathbf{G}_{m,k}^{\left|J \right|-1})] \in K_0(\mathbf{SH}_{\mathfrak{M},ct}(X_{\sigma})).$$


#145
Zaraki

Zaraki

    PQT

  • Phó Quản lý Toán Cao cấp
  • 4273 Bài viết

Bài 55 : Với $n = \prod_{i=1}^{k}p_{i}^{\alpha_{i}}$ trong đó $p_{i} \in P$ và khác nhau . Đặt $f(n)=\prod_{i=1}^{k}\alpha_{i}p_{i}^{\alpha_{i}-1}$ . Chứng minh tồn tại vô hạn $n$ mà $f(n)=f(n-1)+1$ .
Dựa trên ý tưởng của Bằng, mình xin phép được sửa đổi tí cho dễ hiểu - Ego.

Lời giải. Ta sẽ chứng minh tồn tại vô số số nguyên dương $n=3^3 \cdot p_1p_2 \cdots p_k=1+13^2 \cdot q_1q_2 \cdots q_m$ với $p_i \ne 3, q_j \ne 13$. Khi đó $f(n)=27, f(n-1)=2 \cdot 13=26$. Hay ta cần chứng minh có vô số số square-free $3 \nmid a$ sao cho $13 \nmid b=\frac{27a-1}{13^2}$ cũng là số square-free. 

 

Xét số $K$ đủ lớn và $3^4 \cdot 13^2 \mid K$. Ta tìm số các số square-free $a$ có thể trong khoảng từ $1$ đến $\frac{13^2K}{27}$. Ta thấy rằng có tất cả $\frac 23 \cdot \frac{12}{13^3} \cdot \frac{13^2K}{27}=\frac{8K}{351}$ số $x$ thoả mãn $3 \nmid x, 13^2 \|27x-1$ như thế trong khoảng $\left [1, \frac{169K}{27} \right]$. Do đó có tối thiểu số các số square-free $a$ nằm trong khoảng này là $$\frac{8K}{351}- \sum_{p \ne 3,13} \left \lfloor \frac{8K}{315p^2} \right \rfloor \ge \frac{8K}{351} \left( 1- \sum_{p \ne 3,13} \frac{1}{p^2} \right)=A.$$

 

Ta thấy rằng với mỗi số square-free $a$ thì có tương ứng một số $b$ thoả $1 \le b \le K$ và $13 \nmid b= \frac{27a-1}{13^2}$. Hay nói cách khác, có tối đa $A$ số $b$ trong $[1,K]$ sao cho $(169b+1)/27$ là số square-free. Ta thấy rằng có $\frac{12}{13} \cdot \frac{1}{27} \cdot K = \frac{12K}{351}$ số $y$ mà $13 \nmid y, y \equiv (-1) \cdot 169^{-1} \pmod{27}$ nên số các số non square-free $b$ tối đa trong khoảng $\left[ 1,K \right]$ là 

$$\sum_{p \ne 3,13}\left \lfloor \frac{12K}{351} \cdot  \frac{1}{p^2} \right \rfloor \le \frac{12K}{351} \sum_{p \ne 3,13} \frac{1}{p^2}=B.$$

 

Ta có $$ A>B  \iff \frac{2}{5}> \sum_{p \ne 3,13}\frac{1}{p^2}.$$

Và để ý rằng $$\sum_{p \ne 3,13}\frac{1}{p^2} < \frac{\pi^2}{6}-1- \frac{1}{3^2}-\frac{1}{4^2}-\frac{1}{6^2}-\frac{1}{8^2}-\frac{1}{9^2}-\frac{1}{10^2}-\frac{1}{12^2}-\frac{1}{13^2}-\frac{1}{14^2}-\frac{1}{15^2}<0.4= \frac 25$$

Như vậy $A>B$, hay điều này đồng nghĩa với tồn tại một số square-free dạng $a$ sao cho $13 \nmid \frac{27a-1}{13^2}$ cũng là số square-free trong khoảng $[1,K]$. Ta làm điều tương tự với khoảng $[K+1,2K]$. Quá trình cứ tiếp tực như thế, ta suy ra tồn tại vô số $n$ sao cho $n=3^3 \cdot p_1 \cdots p_k=1+13^2 \cdot q_1q_2 \cdots q_m$ và các số $n$ như thế sẽ thoả mãn $f(n)=f(n-1)+1=27$.   $\blacksquare$

 

@bangbang1412: Bằng, ông có lời giải khác cho bài này không? Nếu có cách xây dựng số $n$ thoả mãn thì quá đẹp.

 

Bài 56. [Benelux 2016] Cho $n$ là số nguyên dương. Giả sử rằng ước nguyên dương của nó có thể chia thành các cặp sao cho tổng hai số của mỗi cặp chính bằng một số nguyên tố. Chứng minh rằng các số nguyên tố này phân biệt và không số nào trong chúng là ước của $n$.


Bài viết đã được chỉnh sửa nội dung bởi Zaraki: 13-06-2016 - 22:44

Discovery is a child’s privilege. I mean the small child, the child who is not afraid to be wrong, to look silly, to not be serious, and to act differently from everyone else. He is also not afraid that the things he is interested in are in bad taste or turn out to be different from his expectations, from what they should be, or rather he is not afraid of what they actually are. He ignores the silent and flawless consensus that is part of the air we breathe – the consensus of all the people who are, or are reputed to be, reasonable.

 

Grothendieck, Récoltes et Semailles (“Crops and Seeds”). 


#146
bangbang1412

bangbang1412

    Độc cô cầu bại

  • Phó Quản lý Toán Cao cấp
  • 1667 Bài viết

:(  :(  :D  :D Thực ra tôi đâu giải được bài này , mai tôi sẽ đưa link của nó nhưng hiện tại tôi không biết là có lời giải xây dựng được $n$ vì nhìn vào đề ta cũng na ná đoán đc , chỉ có bù trừ thôi .( kiểu xây dựng $1$ đoạn rồi cm nó tồn tại rồi next sang đoạn khác như bài china tst $2015$) tôi hơi bất ngờ vì ông kiếm ddc số $27$ . Trong lúc xem $2$ trận Euro đã làm xong bài $55$ . 

Bài 55 : 

Ta đặt $n=p_{0}^{a_{0}}....p_{t}^{a_{t}}$ trong đó $a_{i}>0$ và $p_{i}$ nguyên tố . Và giả sử $1=d_{1}<d_{2}<...<d_{k}=n$ là tất cả các ước nguyên dương của $n$ .Trước tiện với cặp $(d_{i},d_{j})$ thỏa mãn tổng nó nguyên tố thì ta gọi là ''  đẹp ''  . Dĩ nhiên ta không quan tâm đến trường hợp có $u,v,i$ đôi một khác nhau mà $(d_{i},d_{u}),(d_{i},d_{v})$ cùng đẹp mà chỉ quan tâm tồn tại một cách chia thỏa mãn . Khi đó có $\frac{k}{2}$ cặp suy ra $k$ chẵn và $a_{i}$ lẻ . Tiếp theo $d_{i}+d_{j}$ nguyên tố thì $(d_{i},d_{j})=1$ nghĩa là không thể có hai số chẵn , mặt khác $d_{i}+d_{j}>2$ nguyên tố lẻ nên tồn tại một số chẵn , điều này cho ta thấy rằng $n$ chẵn , đặt $p_{0}=2$ , có $\frac{k}{2}$ cặp vét hét tất cả các ước chẵn của $n$ , mặt khác số ước chẵn của $n$ là số cách lấy $2$ nhân với một ước khác của $n$ lại có $k=\prod_{i=0}^{t}(a_{i}+1)$ nên $2a_{0}\prod_{i>0}(a_{i}+1)=\prod_{i=0}^{t}(a_{i}+1)$ nên $a_{0}=1$ . Gọi $p>2$ là ước nguyên tố của $n$ và $v_{p}(n)=h$ , mỗi một cặp $(d_{i},d_{j})$ chi có nhiều nhất là một số chia hết cho $p$ , nên mỗi cặp có số số chia hết $p$ không vượt quá số số không chia hết $p$ , lại thấy rằng tất cả các cặp vét hết ước của $n$ do đó $h\leq 1$ . Từ đó $n$ là số $square-free$ , lúc này mọi cặp $(d_{i},d_{j})$ có dạng $(k,\frac{n}{k})$ cho ta thấy rằng tổng các cặp đẹp khác nhau . Rõ ràng $k+\frac{n}{k}$ mà là ước của $n$ thì $gcd(k,\frac{n}{k})>1$ vô lý do $n$ là $square-free$. Vậy ta có đpcm .

Bài 56 :   ( Nguồn : Aops ) Tìm tất cả các cặp $(x,y,p)$ nguyên dương và $p$ nguyên tố để $x+y^{p-1}$ và $y+x^{p-1}$ đều là lũy thừa của $p$ . Bây giờ mới ngủ dậy =)) 


Bài viết đã được chỉnh sửa nội dung bởi bangbang1412: 15-06-2016 - 10:53

$$[\Psi_f(\mathbb{1}_{X_{\eta}}) ] = \sum_{\varnothing \neq J} (-1)^{\left|J \right|-1} [\mathrm{M}_{X_{\sigma},c}^{\vee}(\widetilde{D}_J^{\circ} \times_k \mathbf{G}_{m,k}^{\left|J \right|-1})] \in K_0(\mathbf{SH}_{\mathfrak{M},ct}(X_{\sigma})).$$


#147
Long Phi

Long Phi

    Binh nhất

  • Thành viên
  • 26 Bài viết

- $z$ là gì?

- mấy ngày không onl sao topic ngập tràn analytic thế này


Bài viết đã được chỉnh sửa nội dung bởi Long Phi: 14-06-2016 - 23:45


#148
canhhoang30011999

canhhoang30011999

    Thiếu úy

  • Thành viên
  • 634 Bài viết

Lời giải bài 56 

Xét với $p=2$ thì phương trình có nghiệm $x=k,y=2^{t}-k$ với $t$ là số sao cho $2^{t}>k$

xét với $p>2$ khi đó đặt

$x+y^{p-1}=p^{t}$ (1)

$y+x^{p-1}=p^{k}$  (2)

Giả sử $x \geq y$ thì khi đó $k \geq t$ 

Ta có $y^{(p-1)^{2}}-x^{p-1} $ chia hết có $y^{p-1}+x$ chia hết cho $p^{t}$

nên $y^{(p-1)^{2}}+y-p^{k}$ chia hết cho $p^{t}$

Từ đó suy ra $y(y^{p(p-2)}+1)$ chia hết cho $p^{t}$

Lại có nếu $y$ chia hết cho $p$ thì đặt $v_{p}(y)=t>0$ thì khi đó $v_{p}(x)=(p-1)t$ do (1) (Vô lí do (2))

Vậy $y$ không chia hết cho $p$

Tượng tự $x$ không chia hết cho $p$

Từ đó suy ra $y \equiv -1 (\bmod p)$

Áp dụng định lý LTE ta có $v_{p}(y^{p(p-2)}+1)=v_{p}(y+1)+v_{p}(p(p-2))=v_p{y+1}+1 \geq t$

Từ đó suy ra $y \geq p^{t-1}-1$ hay $p^{t}>(p^{t-1}-1)^{p-1}$

Với $t=1$ thì ta có $y=1$ do $2^{p-1}>p$ nên $p/ y+1=2 $(vô lý)

Với $t>1$ thì ta có $t>(p-1)(t-2)$ nên ta có $t=2$ từ đó suy ra $p^{2}>(p-1)^{p-1}$

nếu $p \geq 5$ thì $(p-1)^{p-1}>(p-1)^{3}>p^{2}$ (vô lý)

Vậy $p=3$ hay $x+y^{2}=9$ nên $x=5$ $y=3$ thỏa mãn 

Vậy bài toán có nghiệm $p=2$ $x=k$ $y=2^{t}-k$ với t là số thỏa mãn $2^{t}>k$ $x=5,y=2,p=3$ $x=2,y=5,p=3$


Bài viết đã được chỉnh sửa nội dung bởi canhhoang30011999: 15-06-2016 - 09:02


#149
canhhoang30011999

canhhoang30011999

    Thiếu úy

  • Thành viên
  • 634 Bài viết

Bài 57 Cho $a_{1}=\frac{2}{3}$,$a_{n+1}=\frac{1}{2}(a_{n}+\frac{1}{3a_{n}}$

CMR $\frac{3}{3a_{n}^{2}-1}$ là số chính phương với mọi $n\in \mathbb{N}^{*}$


Bài viết đã được chỉnh sửa nội dung bởi canhhoang30011999: 15-06-2016 - 09:10


#150
the unknown

the unknown

    Thượng sĩ

  • Thành viên
  • 208 Bài viết

Bài 57 Cho $a_{1}=\frac{2}{3}$,$a_{n+1}=\frac{1}{2}(a_{n}+\frac{1}{3a_{n}}$

CMR $\frac{3}{3a_{n}^{2}-1}$ là số chính phương với mọi $n\in \mathbb{N}^{*}$

Bằng quy nạp ta chứng minh rằng $\frac{3}{3a_n^2-1}$ là số chính phương với mọi $n\in \mathbb{N}^{*}$. Thật vậy theo giả thiết ta có điều cần chứng minh hiển đúng với $n=1$. Giả sử rằng với $n\geq 1$ thì điều giả sử là đúng thì ta sẽ chứng minh cho trường hợp $n+1$.

Theo giả thiết ta có: $\frac{1}{a_n-a_{n+1}}=\frac{6a_n}{3a_n^2-1}$. Theo giả thiết quy nạp ta có: $\frac{3}{3a_n^2-1}$ là số chính phương nên có số $k$ nguyên dương để $\frac{3}{3a_n^2-1}=k^2$$\Rightarrow a_n^2=\frac{k^2+3}{3k^2}\Rightarrow a_n=\sqrt{\frac{k^2+3}{3k^2}}$.

Mặt khác dễ thấy $a_n$ là số hữu tỉ với mọi số nguyên dương $n$, do đó $\sqrt{3(k^2+3)}=3a_nk$ là một số hữu tỉ mà $3(k^2+3)$ là một số nguyên dương nên $3(k^2+3)$ là một số chính phương. Mặt khác $3\mid 3(k^2+3)\Rightarrow 9\mid 3(k^2+3)\Rightarrow 3\mid k^2+3$ và suy ra $\sqrt{\frac{k^2+3}{3}}=\frac{\sqrt{3(k^2+3)}}{3}$ là một số nguyên.

Vậy ta có $\frac{1}{a_n-a_{n+1}}=\frac{6a_n}{3a_n^2-1}=2a_n.k^2=2k\sqrt{\frac{k^2+3}{3}}$ là một số nguyên.

Mặt khác từ giả thiết ta có $6a_na_{n+1}=3a_n^2+1\Leftrightarrow 3(a_n-a_{n+1})^2=3a_{n+1}^2-1\Rightarrow \frac{3}{3a_{n+1}^2-1}=(\frac{1}{a_n-a_{n+1}})^2$ là một số chính phương. Do đó theo nguyên lí quy nạp, bài toán được chứng minh.

Bài 58: (IMO 1987)

Cho $n$ là một số nguyên, $n\geq 2$. Chứng minh rằng nếu $k^2+k+n$ là một số nguyên tố với mọi số nguyên $k$ thỏa mãn $0\leq k\leq \sqrt{\frac{n}{3}}$ thì $k^2+k+n$ cũng sẽ là một số nguyên tố với mọi số nguyên $k$ thỏa $0\leq k \leq n-2$.


Bài viết đã được chỉnh sửa nội dung bởi the unknown: 15-06-2016 - 16:52

$\texttt{If you don't know where you are going, any road will get you there}$


#151
canhhoang30011999

canhhoang30011999

    Thiếu úy

  • Thành viên
  • 634 Bài viết

Lời giải bài 58 

Gọi $p$ là ước nguyên tố nhỏ nhất của các số $k^{2}+k+n$ là hợp số với $\sqrt{\frac{n}{3}} \leq k \leq n-2$

Khi đó ta có $(n-2)^{2}+n-2+n \geq i^{2}+i+n \geq p^{2}$

$=> (n+1)^{2}+1>p^{2} => n+1 \geq p$

Ta gọi số $i$ thỏa điều kiện $i^{2}+i+n$ chia hết cho $p$ với $\sqrt{\frac{n}{3}} \leq i \leq n-2$ là đẹp.

Ta sẽ chứng minh tồn tại $t$ đẹp thỏa $t \leq \frac{p-1}{2}$

Thật vậy xét k đẹp bất kì ta có $k=pt+r$ với $0 \leq r \leq p-1$ thì $r^{2}+r+n$ chia hết cho $p$

Nếu $r \geq \frac{p-1}{2}$ thì xét $(p-r-1)^{2}+p-r-1+n$ chia  hết cho $p$

Vậy tồn tại $t \leq \frac{p-1}{2}$ sao cho $t^{2}+t+n$ chia hết cho $p$

Mà $t^{2}+t+n>p$ nên $t^{2}+t+n \geq p^{2}$

Lại có $ t \geq \sqrt{\frac{n}{3}}$ nên $n \leq \frac{3(n-1)^{2}}{4}$ 

Từ đó $p^{2} \leq t^{2}+t+n \leq \frac{(p-1)^{2}}{4}+\frac{p-1}{2}+\frac{3(p-1)^{2}}{4}=p^{2}-\frac{3p}{2}+\frac{1}{2}$ (Vô lý)

Vậy ta có đpcm

Bài 59 Cho $a_{1}=1;a_{n+1}=a_{n}^{4}-a_{n}^{3}+2a_{n}^{2}+1,n \geq 1$

CMR tồn tại vô hạn số nguyên tố p tm ko có số hạng $a_n$ nào là bội của $p$


Bài viết đã được chỉnh sửa nội dung bởi canhhoang30011999: 15-06-2016 - 22:59


#152
canhhoang30011999

canhhoang30011999

    Thiếu úy

  • Thành viên
  • 634 Bài viết

Do đã lâu rồi bài 59 vẫn chưa có người giải nên mình up bài 60 vậy

Cho dãy $a_{0}=1,a_{1}=1,a_{n+2}=98a_{n+1}-a_{n}-16$

CMR $a_{n}$ là CP với mọi $n \in \mathbb{N}^{*}$


Bài viết đã được chỉnh sửa nội dung bởi canhhoang30011999: 21-06-2016 - 20:37


#153
IMOer

IMOer

    Hạ sĩ

  • Thành viên
  • 50 Bài viết

Bài 59 Cho $a_{1}=1;a_{n+1}=a_{n}^{4}-a_{n}^{3}+2a_{n}^{2}+1,n \geq 1$

CMR tồn tại vô hạn số nguyên tố p tm ko có số hạng $a_n$ nào là bội của $p$

Lâu lắm không vào, thấy bài cũng ngon ngon mà không ai xơi.

 

Lời giải bài 59:

 

Dễ dàng chứng minh được bằng quy nạp rằng: $a_n$ là dãy tăng ngặt và $a_{n+k}\equiv a_k\ (\bmod a_n)$ với mọi $k\in\mathbb{Z}^+$          (1).

Theo bài ra ta có: $a_{n+1}-a_n=(a_n^2+1)(a_n^2-a_n+1)$

Lấy $p$ là một ước nguyên tố nào đó của $a_n^2+1$.

Vì $a_{n + 1}\equiv a_n\ (\bmod p)$, nên $p|a_{n+1}^2+1$.

Suy ra với mọi $k\ge n$ thì $p|a_k^2+1$, nên $p$ không là ước của $a_k$        (2).

Giả sử tồn tại $k<n$ sao cho $p|a_k$.

Theo (1) thì $p|a_{km}$ với mọi $m\in\mathbb{N}$, mâu thuẫn với (2).

Vậy $p$ không là ước của số hạng nào trong dãy $a_n$.

Ta sẽ chứng minh tồn tại vô số số nguyên tố $p$ là ước của một số có dạng $a_n^2+1$.

Ta có:

$\begin{aligned}(a_{n+1}^2+1)-(a_n^2+1)&=(a_n^2+1)(a_n^2-a_n+1)(a_{n+1}+a_n)\\ \Rightarrow a_{n+1}^2+1&=(a_n^2+1)\left(a_n^2-a_n+1)(a_{n+1}+a_n)+1\right)\\&=(a_n^2+1)\left((a_n^2+1)(a_{n+1}+a_n)-a_n(a_{n+1}-a_n)+1-2a_n^2\right)\end{aligned}$

Vì $\gcd(a_n^2+1,1-2a_n^2)=1$ và $a_n^2+1|a_{n+1}-a_n$ nên $\gcd(a_n^2+1,(a_n^2+1)(a_{n+1}+a_n)-a_n(a_{n+1}-a_n)+1-2a_n^2)=1$, suy ra tồn tại số nguyên tố $p$ là ước của $a_{n+1}^2+1$ mà không phải là ước của $a_n^2+1$.

Dễ thấy khi đó $p$ không là ước của $a_k^2+1$ với mọi $k<n$.

Dẫn tới tồn tại vô số số nguyên tố $p$ là ước của một số có dạng $a_n^2+1$.



#154
IMOer

IMOer

    Hạ sĩ

  • Thành viên
  • 50 Bài viết

Do đã lâu rồi bài 59 vẫn chưa có người giải nên mình up bài 60 vậy

Cho dãy $a_{0}=1,a_{1}=1,a_{n+2}=98a_{n+1}-a_{n}-16$

CMR $a_{n}$ là CP với mọi $n \in \mathbb{N}^{*}$

Lời giải bài 60:

 

Xét dãy: $b_0=1, b_1=1, b_{n+2}=10b_{n+1}-b_n$.

Ta có:

$\begin{aligned}2b_{n+1}^2-b_nb_{n+2}&=2b_{n+1}(10b_n-b_{n-1})-b_n(10b_{n+1}-b_n)\\&=10b_nb_{n+1}-2b_{n+1}b_{n-1}+b_n^2\\&=2b_n^2+b_{n+1}(10b_{n+1}-b_n)-2b_{n+1}b_{n-1}\\&=2b_n^2-b_{n-1}b_{n+1} \end{aligned}$

Dẫn tới: $2b_{n+1}^2-b_nb_{n+2}=2b_1^2-b_0b_2=-8$, với mọi $n\in\mathbb{N}$.

Lại có:

$\begin{aligned}b_{n+2}^2&=100b_{n+1}^2-20b_{n+1}b_n+b_n^2\\&= 96b_{n+1}^2-b_n^2-16+(4b_{n+1}^2-20b_{n+1}b_n+2b_n^2+16)\\&= 96b_{n+1}^2-b_n^2-16+2(2b_{n+1}^2-b_nb_{n+2}+8)\\&= 96b_{n+1}^2-b_n^2-16\end{aligned}$

Từ đó suy ra: $a_n=b_n^2$ với mọi $n\in\mathbb{N}$.

 

Bài 61:

Tìm tất cả các bộ số nguyên dương $(a,b,c,d)$ sao cho nếu số nguyên dương $n$ có tất cả các ước nguyên tố lớn hơn 2016 thì $n+d$ là ước của $n+a^n+b^n+c^n$.


Bài viết đã được chỉnh sửa nội dung bởi IMOer: 23-06-2016 - 21:20


#155
canhhoang30011999

canhhoang30011999

    Thiếu úy

  • Thành viên
  • 634 Bài viết

Lời giải bài 61:

Gọi các số nguyên tố bé hơn 2016 lần lượt là $p_{1},p_{2},...p_{k}$ 

theo nguyên tắc Đi-rich-lê thì tồn tại vô số số nguyên tố dạng $2p_{1}p_{2}..p_{k}t +p_{2}p_{3}..p_{k}+2$ và vô hạn số nguyên tố dạng $2p_{1}p_{2}..p_{k}t +p_{2}^{2}p_{3}..p_{k}+2$  

Khi đó tồn tại vô số $p$ là 1 số nguyên tố bất kì là 1 trong 2 dạng trên thỏa mãn $\frac{p-1}{2}$ lẻ

theo định lí Trung Quốc về phần dư thì tồn tại n sao cho

$n\equiv 0 (\bmod p), \\ n\equiv 0(mod \frac{p-1}{2})\\ n\equiv 1(mod 2016!)$

Khi đó $a^{n} \equiv +1,-1 (mod p)$

nên $p/d-3$ hoặc $d-1$ hoặc $d+1$ hoặc $d+3$

do có vô số $p$ nên $d=1$ hoặc $d=3$ hoặc $d=-1$ (loại) hoặc $d=-3$ (loại)

nếu d=1 thì theo Đi-rich-lê thì tồn tại vô số nguyên tố q dạng $p_{1}p_{2}..p{k}t+1$ khi đó chọn $n=2q-1$ thì ta có $a+b+c-1$ chia hết cho $q$

$=>a+b+c=1$ (vô lý)

Nếu $d=3$ thì chọn $n=4q-3$ thì thì $a+b+c-3$ chia hết cho $q$ nên $a+b+c=3 $ do đó $a=b=c=1$


Bài viết đã được chỉnh sửa nội dung bởi canhhoang30011999: 27-06-2016 - 14:55


#156
canhhoang30011999

canhhoang30011999

    Thiếu úy

  • Thành viên
  • 634 Bài viết

Bài 62 Tìm a,b,c nguyên dương sao cho

$a^{2}+b^{2}+c^{2}$ chia hết cho $2013(ab+bc+ca)$

nguồn (Iran TST 2013)

P/s:Anh IMOer có thể đăng cách giải bài 61 của anh được không ạ


Bài viết đã được chỉnh sửa nội dung bởi canhhoang30011999: 26-06-2016 - 15:25


#157
viet nam in my heart

viet nam in my heart

    Thượng sĩ

  • Điều hành viên OLYMPIC
  • 242 Bài viết

Bài 62 Tìm a,b,c nguyên dương sao cho

$a^{2}+b^{2}+c^{2}$ chia hết cho $2013(ab+bc+ca)$

nguồn (Iran TST 2013)

Lời giải bài 62

Ta chứng minh không tồn tại bộ $3$ số $a,b,c$ nguyên dương thỏa mãn $a^2+b^2+c^2=3n\left(ab+bc+ca\right)$ với mọi số nguyên dương $n$

Giả sử tồn tại bộ $3$ số $a,b,c$ nguyên dương thỏa mãn $a^2+b^2+c^2=3n\left(ab+bc+ca\right)$

Chọn bộ $(x,y,z)$ thỏa mãn tổng 3 số nhỏ nhất trong các bộ thỏa mãn

Ta có: $x^2+y^2+z^2=3n\left(xy+yz+zx\right)$ suy ra $\left(x+y+z\right)^2=\left(3n+2\right)\left(xy+yz+zx\right)$

Tồn tại số nguyên tố $p, p \equiv 2 (mod3)$ , $p$\ $3n+2,v_p{(3n+2)}$ là một số lẻ

Suy ra $x+y+z \vdots p$ và $v_p{(x+y+z)^2}$ là một số chẵn 

Từ đó $xy+yz+zx \vdots p \Rightarrow x^2+y^2+z^2 \vdots p \Rightarrow (x^2+y^2+z^2+xy+yz+zx)-x\left(x+y+z\right) \vdots p \Rightarrow y^2+yz+z^2 \vdots p$ 

Theo một bổ đề quen thuộc thì với số nguyên tố $p \equiv 2 (mod3)$ ta có $y^2+yz+z^2 \vdots p \Leftrightarrow \left\{\begin{matrix} x \vdots p\\ y \vdots p \end{matrix}\right.$

Mà $x+y+z \vdots p$ nên $x,y,z \vdots p$

Khi đó lấy $x_1=\dfrac{x}{p},y_1=\dfrac{y}{p},z_1=\dfrac{z}{p}$ thì $x_{1}^2+y_{1}^2+z_{1}^2=3n(x_1y_1+y_1z_1+z_1x_1)$ nhưng $x_1+y_1+z_1 < x+y+z$ (vô lý)

Do đó không tồn tại các số $a,b,c$ nguyên dương thỏa mãn $a^2+b^2+c^2$ chia hết cho $2013(ab+bc+ca)$


Bài viết đã được chỉnh sửa nội dung bởi viet nam in my heart: 26-06-2016 - 14:24

"Nếu bạn hỏi một người giỏi trượt băng làm sao để thành công, anh ta sẽ nói với bạn: ngã, đứng dậy là thành công." Isaac Newton

VMF's Marathon Hình học Olympic


#158
viet nam in my heart

viet nam in my heart

    Thượng sĩ

  • Điều hành viên OLYMPIC
  • 242 Bài viết

Bài 63: Cho $p= 4^n+1$ với $n$ là một số nguyên dương. Chứng minh rằng : $p$ là một số nguyên tố $\Leftrightarrow 3^{\dfrac{p-1}{2}}+1 \vdots p$


"Nếu bạn hỏi một người giỏi trượt băng làm sao để thành công, anh ta sẽ nói với bạn: ngã, đứng dậy là thành công." Isaac Newton

VMF's Marathon Hình học Olympic


#159
IMOer

IMOer

    Hạ sĩ

  • Thành viên
  • 50 Bài viết

Lời giải bài 63:

 

*) Với $3^{\frac{p-1}{2}}+1\equiv0\ (\bmod{p})$ thì $3^{\frac{p-1}{2}}\equiv -1\ (\bmod{p})$.

Gọi $d$ là cấp của 3 theo modulo $p$, khi đó: $d\nmid \dfrac{p-1}{2}$.

Mà $3^{p-1}\equiv 1\ (\bmod{p})$ nên $d\mid p-1$, dẫn tới: $d=p-1$. Từ đó suy ra: $p$ là số nguyên tố.

 

*) Với $p$ là số nguyên tố, ta có: $p\equiv 2\ (\bmod{3})$ và $p\equiv 1\ (\bmod{4})$.

Ta có: $\left(\dfrac{3}{p}\right)=\left(\dfrac{p}{3}\right)=\left(\dfrac{2}{3}\right)=-1$.

Nên: $3^{\frac{p-1}{2}}\equiv\left(\dfrac{3}{p}\right)\equiv -1\ (\bmod{p})$.

 

Bài 64: Tìm tất cả các hàm số $f:\mathbb{Z}^+\to\mathbb{Z}^+$ thoả mãn đồng thời:

(i) $f(n)\ge n$ với mọi $n\in\mathbb{Z}^+$.

(ii) $f(m+n)\mid f(m)+f(n)$ với mọi $m,n\in\mathbb{Z}^+$.



#160
viet nam in my heart

viet nam in my heart

    Thượng sĩ

  • Điều hành viên OLYMPIC
  • 242 Bài viết

Lời giải bài 63:

Khi đó: $d\nmid \dfrac{p-1}{2}$.

Mà $3^{p-1}\equiv 1\ (\bmod{p})$ nên $d\mid p-1$, dẫn tới: $d=p-1$. Từ đó suy ra: $p$ là số nguyên tố.

 

Anh làm hơi vội đoạn này rồi ạ ví dụ $p=61$ và $d=12$

Chỗ đó chỉ suy ra $v_2{(d)}=v_2{(p-1)}$ thôi ạ


"Nếu bạn hỏi một người giỏi trượt băng làm sao để thành công, anh ta sẽ nói với bạn: ngã, đứng dậy là thành công." Isaac Newton

VMF's Marathon Hình học Olympic





1 người đang xem chủ đề

0 thành viên, 1 khách, 0 thành viên ẩn danh